Question

2. Consider a spin-1/2 particle. The physical quantity K is represented by the operator (written in the S-basis): k=ko –2i) *
0 0
Add a comment Improve this question Transcribed image text
Answer #1

-2i (+zki 0 10 -2k6 | KEK (tai 0 - (+261 0 1. K is a real number with the appropriate dimensions. .@ Eigenvalues are the roosThen - 2K 2 km any (out) - Genealogy 2k22 on :.. -2kid=244 ① . and kine=2kl2 ☺ from Ø _in=4 and from h i = xz . These two equ:Non maliped egenstate, x, s as for any nonmalised Xi =1 l ) state we matos, haveSimilarly i let to eigenvalue Then me = -24 eigenstac connes ponding be X2 = 1 -LK/ - R X2 = 22 X2 O, 10 -zki, ki exi 01 or /of s in superposition © The parice could be in any one of the states x, and X2 atten the measurement of ů. o As initially isePutring ruese values in see that [Sz &] to. of spin (st) and ů are measurable. equation ③ we Henced z-compment not simultaneo

Add a comment
Know the answer?
Add Answer to:
2. Consider a spin-1/2 particle. The physical quantity K is represented by the operator (written in...
Your Answer:

Post as a guest

Your Name:

What's your source?

Earn Coins

Coins can be redeemed for fabulous gifts.

Not the answer you're looking for? Ask your own homework help question. Our experts will answer your question WITHIN MINUTES for Free.
Similar Homework Help Questions
  • [3] A spin-1/2 particle is in the state IW) 1/311) +i2/3|). (a) A measurement is made of the x component of the spi...

    [3] A spin-1/2 particle is in the state IW) 1/311) +i2/3|). (a) A measurement is made of the x component of the spin. What is the probability that the spin will be in the +z direction? (b) Suppose a measurement is made of the spin in the z direction and it is found that the particle has m,#1/2. what is the state after the measurement? (c) Now a second measurement is made immediately after to determine the spin in the...

  • 2. Spin-1/2 system: (20 points) The Pauli matrices are, 0 -1 from which we can define...

    2. Spin-1/2 system: (20 points) The Pauli matrices are, 0 -1 from which we can define the spin matrices, s.-슬&z, Šv = , S.-출.. We'll use the eigenkets of S that, for the spin half system, they can be represented by the spinors, a) Show, by matrix multiplication that |+) and |-) are eigenstates of the S operator and determine the eigenvalues. Show that they are not eigenstates of S and Sy b) Show that the matrix squares s ,...

  • Consider the state of a spin-1/2 particle 14) = v1o (31+z) + i] – z)) where...

    Consider the state of a spin-1/2 particle 14) = v1o (31+z) + i] – z)) where | z) are the eigenstates of the operator of the spin z-component $z. 1. Show that [V) is properly normalized, i.e. (W14) = 1. 2. Calculate the probability that a measurement of $x = 6x yields 3. Calculate the expectation value (Šx) for the state 14) and its dispersion ASx = V(@z) – ($()2. 4. Assume that the spin is placed in the magnetic...

  • A spin-1 particle interacts with an external magnetic field B = B. The interaction Hamiltonian for the system is H = gB-S, where S-Si + Sỳ + SE is the spin operator. (Ignore all degrees of freedom ot...

    A spin-1 particle interacts with an external magnetic field B = B. The interaction Hamiltonian for the system is H = gB-S, where S-Si + Sỳ + SE is the spin operator. (Ignore all degrees of freedom other than spin.) (a) Find the spin matrices in the basis of the S. S eigenstates, |s, m)) . (Hint: Use the ladder operators, S -S, iS, and S_-S-iS,, and show first that s_ | 1,0-ћ /2 | 1.-1)) . Then use these...

  • 3. (a) A particle orbits a fixed point in space. Given that L-=-ih- , find the...

    3. (a) A particle orbits a fixed point in space. Given that L-=-ih- , find the eigenvalues аф and normalised eigenfunctions of the z-component of the particle's angular momentum. (5 marks) Explain clearly your reasoning at each important step An electron is in the spin state x)-A G) Normalise x) and hence determine the constant A (ii) If you measured the z-component of the electron's spin, s, . what values could (b) in the usual s basis you get and...

  • Incorrect Question 9 0/1 pts The state % = to 4-1 + tzv. + tag 42...

    Incorrect Question 9 0/1 pts The state % = to 4-1 + tzv. + tag 42 is a linear combination of three orthonormal eigenstates of the operator Ô corresponding to eigenvalues - 1, 1, and 2. What is the expectation value for Ô for this state? V o 13+2821 Which of the following is true of observable in quantum mechanics? I. They are represented by Hermitian operators. II. Multiple observables can never be simultaneously measured. III. The operators representing observables...

  • (L43*) Spin can be represented by matrices. Show that all three spin matrices l 0 2 0 -1 0),"2=2 1 have eigenval...

    (L43*) Spin can be represented by matrices. Show that all three spin matrices l 0 2 0 -1 0),"2=2 1 have eigenvalues of +1/2h and -1/2h. Calculate the corresponding eigenfunctions which we will denote as α-and β-eigenfunctions corresponding to spin l/2 particles. Show that Sj can be determined by the commutation of the other two matrices sn and sm, n, maj. Prove that the (2×2) matrix sz-s' +ss+s, commutes with all spin matrices, ie. s2s,-sis-. Calculate the eigenvalues of s2....

  • 4. 10 points The Spin operators for a spin-1/2 particle can be described by the Pauli...

    4. 10 points The Spin operators for a spin-1/2 particle can be described by the Pauli matrices: 0 1 0 0 ,02= 0 -1 1 ¿ a) Write the normalized eigenvectors of Oz, I+) and 1-) which are defined such that 0z|+) = 1+) and 0z1-) = -1-), as column vectors in the same basis as the Pauli matrices given above. (You can assume without loss of generality that these eigenvectors are real.) (3 pts) b) Consider an eigenvector (V)...

  • 2. Addition of Angular Momentum a) (8pts) Given two spin 1/2 particles, what are the four possibilities for their spin configuration? Put your answer in terms of states such as | 11). where the f...

    2. Addition of Angular Momentum a) (8pts) Given two spin 1/2 particles, what are the four possibilities for their spin configuration? Put your answer in terms of states such as | 11). where the first arrow denotes the z-component of the particle's spin. Identify the m values for each state. b)(7pts) If you apply the lowering operator to a state you get Apply the two-state lowering operator S--S(,) +S(), where sti) acts on the first state and S acts on...

  • Two particle state

    Two Particle State problem, measurement of total angular momentum, finding probabilities, action of lowering operatora) A particle of spin \(3 / 2\) and a particle of spin 1 are found in the state \(\left|\frac{3}{2}-\frac{1}{2}\right\rangle\) of total spin. Assume that the particles are in a state of zero orbital angular momentum.Find which values of the z-component of the spin of the two particles can be measured and with what probabilities.The following information applies to parts \(\mathbf{b}\) ) and \(\mathbf{c}\) ).An electron...

ADVERTISEMENT
Free Homework Help App
Download From Google Play
Scan Your Homework
to Get Instant Free Answers
Need Online Homework Help?
Ask a Question
Get Answers For Free
Most questions answered within 3 hours.
ADVERTISEMENT
ADVERTISEMENT
ADVERTISEMENT